Arc from Car

Logic Level 2

C A R C A + C A R C \begin{array} {cccc} \large & & \color{#D61F06}C & \color{#3D99F6}A & \color{#20A900}R \\ \large & & & \color{#D61F06}C & \color{#3D99F6}A \\ \large + & & & & \color{#D61F06}C \\ \hline \large & & \color{#3D99F6}A & \color{#20A900}R & \color{#D61F06}C \end{array}

If C \color{#D61F06}C , A \color{#3D99F6}A , and R \color{#20A900}R are (not necessarily distinct) non-zero digits, what is the 3-digit final sum A R C ? \overline{ {\color{#3D99F6}A} {\color{#20A900}R} {\color{#D61F06}C } }?


The answer is 373.

This section requires Javascript.
You are seeing this because something didn't load right. We suggest you, (a) try refreshing the page, (b) enabling javascript if it is disabled on your browser and, finally, (c) loading the non-javascript version of this page . We're sorry about the hassle.

7 solutions

Chew-Seong Cheong
Jul 25, 2017

The sum is as follows:

111 C + 11 A + R = 100 A + 10 R + C 110 C = 89 A + 9 R = 80 A + 9 A + 9 R = 80 A + 9 ( A + R ) Last column: R + A + C = 10 + C A + R = 10 = 80 A + 90 11 C = 8 A + 9 11 C 9 = 8 A Since 8 A is even and > 0 , C must be odd and > 1 33 9 = 8 A No integer solution other than C = 3 A = 3 R = 10 A = 7 \begin{aligned} 111C + 11A + R & = 100A + 10R + C \\ 110C & = 89A+9R \\ & = 80A + 9A+9R \\ & = 80A + 9({\color{#3D99F6}A+R}) & \small \color{#3D99F6} \text{Last column: } R+A + C = 10+C \implies A+R =10 \\ & = 80A + 90 \\ \implies 11C & = 8A+9 \\ 11{\color{#3D99F6}C} - 9 & = 8A & \small \color{#3D99F6} \text{Since } 8A \text{ is even and }> 0, C \text{ must be odd and }> 1 \\ {\color{#3D99F6}33} - 9 & = 8A & \small \color{#3D99F6} \text{No integer solution other than }C=3 \\ \implies A & = 3 \\ \implies R & = 10-A = 7 \end{aligned}

A R C = 373 \implies \overline{ARC} = \boxed{373} .

why is R + A + C = 10 + C ? why not other multiples of ten?

Terrell Bombb - 3 years, 10 months ago

Log in to reply

The last column \(\begin{array} {} & R \\ & A \\ + & C \\ \hline 1 & C \end{array}\)

Since A , C 0 A, C \ne 0 , this is the only answer.

Chew-Seong Cheong - 3 years, 10 months ago

R + A > 0 R+A>0 as A > 0 A>0 and R > 0 R>0 . As A < 10 A<10 and R < 10 R<10 , A + R < 20 A+R<20 which means A + R = 10 A+R = 10 .

Tarmo Taipale - 3 years, 10 months ago

Log in to reply

But we are adding C too!

Rakibul hossain - 3 years, 8 months ago

Log in to reply

@Rakibul Hossain Let us have an integer k k so that R + A + C = 10 k + C R+A+C=10k+C . By subtracting C from both sides, we get R + A = 10 k R+A = 10k . A A and R R are non-zero digits (integers between 1 and 9) which means 1 A 9 1\leq A\leq9 and 1 R 9 1\leq R\leq9 , so we get 2 A + R 18 2\leq A+R\leq18 . Due to this, 1 is the only possible value for k k .

Tarmo Taipale - 3 years, 8 months ago

A nice and different approach.

Tarmo Taipale - 3 years, 10 months ago

Log in to reply

Glad that you like it.

Chew-Seong Cheong - 3 years, 10 months ago

In the 3rd step it appeared initially that you are substituting A+R for R. I see now that you moved over some As to exploit the knowledge you had about A+R=10. Fancy! But maybe give us dummies a break and explain that step.

Kevin Jaques - 3 years, 10 months ago

Log in to reply

Thanks, I added a line to clarify that.

Chew-Seong Cheong - 3 years, 10 months ago

I like these types of problems. Any idea on what they're called so I could search up some more?

Matthew Kulec - 3 years, 10 months ago

Log in to reply

Look under the topic Logic. This type of problems are call cryptograms.

Chew-Seong Cheong - 3 years, 10 months ago

@Chew-Seong Cheong Thanks!

Matthew Kulec - 3 years, 10 months ago

How u got the 1st line eqn

Mahb Rock - 3 years, 8 months ago

Log in to reply

C A R + C A + C = A R C \overline {CAR}+ \overline {CA} + \overline C = \overline {ARC} gives ( 100 C + 10 A + R ) + ( 10 C + A ) + C = 100 A + 10 R + C (100C+10A+R)+(10C+A)+C=100A+10R+C which gives the first line.

Tarmo Taipale - 3 years, 8 months ago
Tarmo Taipale
Jul 24, 2017

From the ones column, we get A + R = 10 A + R = 10 . Now 1 is the carrying digit of ones row.

Thus, we get from the tens column A + C + 1 = R A + C + 1 = R or A + C + 1 = 10 + R A + C + 1 = 10 + R .

From the hundreds column we get:

If A + C + 1 = R A + C + 1 = R , A = C A=C . As A + R = 10 A + R = 10 we get A + C + 1 = 10 A A+C+1 = 10 - A and 2 A + C + 1 = 10 2A + C + 1 = 10

If A + C + 1 = 10 + R A + C + 1 = 10 + R , A = C + 1 A = C+1 . As A + R = 10 A + R = 10 we get A + C + 1 = 20 A A+C+1 = 20 - A and 2 A + C + 1 = 20 2A + C + 1 = 20

This means, either of the following statements holds true:

A = C A = C and 3 A + 1 = 10 3A + 1 = 10

OR

A = C + 1 A = C+1 and 3 A = 20 3A = 20

As 3 doesn't divide 20, there's no solution to the second case. In the first case, we get A = 3 A=3 and thus C = 3 C=3

From A + R = 10 A+R=10 we get R + 3 = 10 R+3 = 10 and R = 7 R=7 .

By checking, we get 337 + 33 + 3 = 373 337+33+3 = 373 . As C = 3 , A = 3 , R = 7 C=3, A=3, R=7 is the only possible solution, the answer is 373 \boxed{373} .

It has always been tacitly assumed that different letters stood for different numbers. In this context, there is no solution to this problem. Ed Gray

Edwin Gray - 3 years, 10 months ago

Log in to reply

When problems like this require the letters to have distinct values, that requirement is made explicit. "Tacit assumptions" have no place in mathematics.

Richard Desper - 3 years, 10 months ago

Log in to reply

True. I made that assumption at first and followed it to the mathematical conclusion that it was impossible, ergo, there was a problem with the logic I applied to the problem. At that point, a re-read of the question revealed where I had used biased thinking (i.e. this kind of problems "usually" use different variables) instead of understanding exactly what was asked.

Edgar Lemos - 3 years, 10 months ago

Every time I've seen the assumption of different letters standing for different numbers, it has been mentioned. But I'll fix the problem a bit and mention that there can be two same letters.

EDIT: Oh, I can't edit this problem for some reason. Maybe it's because it's already a part of a problem set.

Tarmo Taipale - 3 years, 10 months ago

Log in to reply

Leave it as it is. It makes what otherwise would be a very straightforward problem a bit trickier.

Edgar Lemos - 3 years, 10 months ago

Tamo, thanks for trying, Ed Gray

Edwin Gray - 3 years, 10 months ago

Agree totally with Edwin Gray.....back to the drawing board Tamo!

Tim Rolph - 3 years, 10 months ago

Log in to reply

Hey, I would like to edit the problem so it would be clear for everyone that the digits can be same. However, I can't edit this problem for some reason, maybe it's because it has been added to a problem set.

Tarmo Taipale - 3 years, 10 months ago

I tend to agree that this problem should state explicitly that the three numbers, represented by three distinct letters, are not necessarily distinct numbers. This problem strikes me as unnecessarily deceptive and misleading. I used all the same reasoning and equations as the solution --BUT with the assumption that the numbers were distinct; and there was no solution in that case.

DeeDee Moran - 3 years, 10 months ago

Log in to reply

I am sorry but it is not the problem maker's business to account for your assumptions to make it as easy as possible for you. I, too, made the assumption at first, but when I realized that it would have no possible solution and re-read the problem, I was made aware of my assumptive thinking and reformulated. That is a /good/ thing and, in fact, I thought that it had been done so on purpose, as this website often promotes logical analysis and lateral thinking in addition to hard maths.

Edgar Lemos - 3 years, 10 months ago

How can you be sure that A+R=10 it can be 20 too!

Rakibul hossain - 3 years, 8 months ago

Log in to reply

No, it can't because A and R are digits (integers) between 1 and 9. So A 9 A\leq9 and B 9 B\leq9 which means A + B 9 + 9 = 18 < 20 A+B\leq 9+9=18<20 .

Tarmo Taipale - 3 years, 8 months ago
Cordell Booth
Aug 1, 2017

This took me awhile to solve. I did not like the fact that they used two different variables for the same number.

I initially thought C + 1 = A C+1=A and A + C + 1 = 10 + R A+C+1=10+R and that R + A = 10 R+A=10

These were my three equations I used and set up a system of equations

The answers I got, after trying multiple methods of solving, were not integers. So I reexamined the equations

I went back and realized that R + A = 10 R+A=10 has to be true.

There is no other way of getting C C in the ones place with C C being one of the digits added to it. As long as the other digits are non-zero

I next went back to reexamine A + C + 1 = 10 + R A+C+1=10+R

A + C + 1 A+C+1 has to be true because since the ones place give us something higher than 10 10 we have to carry the 1 1 over to the tens place

However getting 10 + R 10+R as a result does not have to be the case if C = A C=A

This would mean that A + C + 1 = R A+C+1=R and that nothing carries over to the hundreds place

So now that I modified the equations to: C = A C=A ,

A + C + 1 = R A+C+1=R , and

R + A = 10 R+A=10
the system of equations could be modified to

A + A + 1 = R A+A+1=R

and

R + A = 10 R+A=10

if 2 A + 1 = R 2A+1=R and A + R = 10 A+R=10 then A + ( 2 A + 1 ) = 10 A + (2A+1)=10

3 A + 1 = 10 3A+1=10

3 A = 9 3A=9

A = 3 A=3 , R = 7 R=7 , C = 3 C=3

Nice job on showing the steps of how to figure out the solution. That's really helpful

Agnishom Chattopadhyay - 3 years, 10 months ago

Log in to reply

Yep. The explanation is helpful. Could you clarify where you said A + C + 1 = R +10 unless A = C . That would be true if A and C were greater than or equal to 5. But less than that and R will still be positive?

Sulayman Hussain - 3 years, 10 months ago

Log in to reply

I'm not sure I understand what you are asking in regards to the numbers being positive, however I will try to clarify why I made these assumptions. Since we get a number greater than 10 in the ones place, we must add that 1 in the tens place. That gives us A + C +1. Looking ahead to the hundreds place, C being by itself and A being what it result too, my first assumption was that, if 1 is being carried over from the tens place, then C+1= A must be true. Since C+1 ≠ A, that means nothing carries over from the tens place. This means that when adding all the digits from the tens place, our result has to be < 10. So A + C + 1 = R
I hope that helped.

Cordell Booth - 3 years, 9 months ago

Log in to reply

@Cordell Booth It did. Very many thanks for the clear and terse words. It is truly a talent.

Sulayman Hussain - 3 years, 9 months ago

Yeah, I neither liked that. But I had to make a little mistake to realize C = A

Natalia QuesoSP - 3 years, 10 months ago
Jason Dyer Staff
Aug 2, 2017

From the ones column R + A + C = (10s digit) + C . R + A + C = \text{(10s digit)} + C . Subtracting C C on both sides, R + A = (10s digit) , R + A = \text{(10s digit)} , which implies (since only two digits are being added) that R + A = 10. R + A = 10 .

This means the tens column has a carry of 1.

Suppose C A , C \neq A , so we know there is a carry digit of 1 from the tens column.

Combining this information and subtracting C C from both sides of the addition leaves us with:

1 1 C A R + C A A R 0 \begin{array} {cccc} \large & & 1 & 1 & \\ \large & & \color{#D61F06}C & \color{#3D99F6}A & \color{#20A900}R \\ \large + & & & \color{#D61F06}C & \color{#3D99F6}A \\ \hline \large & & \color{#3D99F6}A & \color{#20A900}R & \color{#69047E}0 \end{array}

This gives us the equations 1 + C = A , 1 + C = A , 1 + A + C = 10 + R , 1 + A + C = 10 + R , and R + A = 10. R + A = 10 . Doing some substitution leads to the equation 2 A = 10 + 10 A 2A = 10 + 10 - A which reduces to 3 A = 20. 3A = 20 . Since A A is an integer, this is impossible, and our supposition that C A C \neq A is incorrect.

So C = A : C = A :

1 C C R + C C C R 0 \begin{array} {cccc} \large & & & 1 & \\ \large & & \color{#D61F06}C & \color{#3D99F6}C & \color{#20A900}R \\ \large + & & & \color{#D61F06}C & \color{#3D99F6}C \\ \hline \large & & \color{#3D99F6}C & \color{#20A900}R & \color{#69047E}0 \end{array}

This implies 1 + C + C = R 1 + C + C = R and R + C = 10. R + C = 10 . Substitution implies 1 + C + C + C = 10 , 1 + C + C + C = 10 , or 3 C = 9. 3C = 9 . So C = 3 , C = 3, A = 3 , A=3, and (using R + C = 10 R + C = 10 again) R = 7. R = 7 .

This is the best solution

Mahb Rock - 3 years, 8 months ago
Skander Mhadhbi
Jul 31, 2017

0 < A , C , R < 10 0<A,C, R <10

From the first column :

  • We have R + A + C < 10 R + A + C < 10 or R + A + C > 10 R + A + C > 10

    • If R + A + C < 10 R + A + C < 10 then : { R + A + C = C A + C = R C = A \left\{\begin{matrix} R + A + C = C \\ A + C = R \\ C = A \end{matrix}\right. 3 A = 0 A = 0 \implies 3A = 0 \implies A = 0

So R + A + C > 10 R + A + C = 10 q + C R + A = 10 q , q = 1 o r q = 2 R + A + C > 10 \implies R + A + C = 10q + C \Leftrightarrow \boxed{R + A = 10q \ , \ q = 1 \ or \ q = 2}

  • Now A + C + q > 10 A + C + q > 10 or A + C + q < 10 A + C +q < 10

    • If A + C + q > 10 A + C + q > 10 Then we would have : A + C + q = 10 q + R , q = 1 A +C + q = 10q' + R, q' = 1 since A < 10 A < 10 and C < 10 C <10 So A + C + q = 10 + R A+C + q =10 + R

{ R + A = 10 q A + C + q = 10 + R C + 1 = A 2 C + q 9 + A = 10 q 3 A = 9 q 11 , { 3 A = 2 i f q = 1 3 A = 7 i f q = 2 \left\{\begin{matrix} R + A = 10q \\ A + C + q =10 + R \\ C + 1= A \end{matrix}\right.\implies 2C + q - 9 + A = 10q \implies 3A = 9q - 11 , \left\{\begin{matrix} 3A = -2 \ if \ q = 1 \\ 3A = 7 \ if \ q = 2 \end{matrix}\right. Impossible

So A + C + q < 10 \boxed{A + C +q < 10}

And we wloud have :

  • { R + A = 10 q A + C + q = R C = A \left\{\begin{matrix} R + A = 10q \\ A + C + q =R \\ C = A \end{matrix}\right. R = 2 A + q A = 9 q \implies R = 2A +q \implies A = 9q

{ 3 A = 9 i f q = 1 A = 3 3 A = 18 i f q = 2 A = 6 C = 6 , R = 20 6 = 14 : i m p o s s i b l e \left\{\begin{matrix} 3A = 9 \ if \ q = 1 \implies \boxed{A = 3} \\ 3A = 18 \ if \ q = 2 \implies A = 6 \implies C = 6, R = 20 - 6 = 14 \ :\ impossible \end{matrix}\right.

So : { R + A = 10 A + C + 1 = R C = A = 3 A = 3 , C = 3 , R = 7 , A R C = 373 \left\{\begin{matrix} R + A = 10 \\ A + C + 1 =R \\ C = A = 3 \end{matrix}\right. \implies \boxed{ A = 3, C = 3, R = 7, ARC = 373 }

Nice to see a solution where all posibilities are covered.

José Carlos Hurtado Romero - 3 years, 10 months ago
Colin Yang
Aug 6, 2017

From the above information, we get three important equations: First: 110C = 89 A + 9R (From 111C + 11A + R = 100A + 10R + C) Second: R+A = 10 or R = 10 - A Thrid: A+C+1=R

From Second and Third, we get: A+C+1 = 10 - A and therefore, C= 9 - 2A.

Now, let's substitute C and R with As and we get:

 110 (9 - 2A) = 89A + 9 (10 - A)

Then we get: 990 - 220A = 89A + 90 - 9A Then we get: 900 = 300A Finally: A = 3, therefore R= 7, therefore, C= 3

I created a python code which checked every possible solution after i couldn't figure it out after 5 minutes of trying. (Yes i gave up quick xD still didnt look at the answer after i got it.) https://pastebin.com/76sRZWV0

1
2
3
4
5
6
7
8
9
for i in range(1,10):
    for j in range(1,10):
        for k in range(1,10):
            car = 100*i+10*j+k
            ca = 10*i+j
            c = i
            arc = 100*j+10*k+i
            if car+ca+c==arc:
                print(arc)

Great job on using programming to solve this puzzle.

Follow Up Question: How would you have managed this if you needed to check for 10 variables instead of 3? Is there a way to generalize the 3 loops?

Agnishom Chattopadhyay - 3 years, 10 months ago

I added the code to the answer since it wasn't long, and people on the mobile app aren't likely to do the work to cut and paste a URL in their browser.

Jason Dyer Staff - 3 years, 10 months ago

Thanks Jason I appreciate it!

A Former Brilliant Member - 3 years, 10 months ago

0 pending reports

×

Problem Loading...

Note Loading...

Set Loading...